ThmDex – An index of mathematical definitions, results, and conjectures.
Result R3568 on D2455: Real geometric mean
Real AM-GM inequality
Formulation 0
Let $x_1, \dots, x_N \in [0, \infty)$ each be an D4767: Unsigned real number.
Then
(1) \begin{equation} \left( \prod_{n = 1}^N x_n \right)^{\frac{1}{N}} \leq \frac{1}{N} \sum_{n = 1}^N x_n \end{equation}
(2) \begin{equation} \left( \prod_{n = 1}^N x_n \right)^{\frac{1}{N}} = \frac{1}{N} \sum_{n = 1}^N x_n \quad \iff \quad x_1 = x_2 = \cdots = x_N \end{equation}
Proofs
Proof 0
Let $x_1, \dots, x_N \in [0, \infty)$ each be an D4767: Unsigned real number.
This result is a particular case of R1557: Weighted real AM-GM inequality with $\lambda_n = 1/N$. $\square$